[Ciągi][Analiza] Niestandardowa granica

Zadania z kółek matematycznych lub obozów przygotowujących do OM. Problemy z minionych olimpiad i konkursów matematycznych.
Regulamin forum
Wszystkie tematy znajdujące się w tym dziale powinny być tagowane tj. posiadać przedrostek postaci [Nierówności], [Planimetria], itp.. Temat może posiadać wiele różnych tagów. Nazwa tematu nie może składać się z samych tagów.
Piotr Rutkowski
Użytkownik
Użytkownik
Posty: 2234
Rejestracja: 26 paź 2006, o 18:08
Płeć: Mężczyzna
Lokalizacja: Warszawa
Podziękował: 22 razy
Pomógł: 390 razy

[Ciągi][Analiza] Niestandardowa granica

Post autor: Piotr Rutkowski »

Udowodnij, że:
\(\displaystyle{ \lim_{n\to } \{\{3^{n}\}\}=+\infty}\), gdzie:
\(\displaystyle{ \{\{x\}\}}\) to suma cyfr liczby x.
Awatar użytkownika
mol_ksiazkowy
Użytkownik
Użytkownik
Posty: 11409
Rejestracja: 9 maja 2006, o 12:35
Płeć: Mężczyzna
Lokalizacja: Kraków
Podziękował: 3155 razy
Pomógł: 748 razy

[Ciągi][Analiza] Niestandardowa granica

Post autor: mol_ksiazkowy »

\(\displaystyle{ \{\{x\}\}}\) to suma cyfr liczby x.
[/quote]ekh, no mozna mocniej , niech a bedzie l. parzysta niepodzielna przez k=5, to wtedy suma cyfr \(\displaystyle{ a^n}\) dazy do + nieskonczonosci. Zapis \(\displaystyle{ a^n}\) dziesietny : \(\displaystyle{ a_1, a_2, ....}\) , (a1 to ostatnia jej cyfra ), itd. jest nie za trudny lemat
Jeśli \(\displaystyle{ 1 \leq j \leq \frac{n}{4}}\) , to co najmniej jedna z cyfr \(\displaystyle{ a_{j+1}, a_{j+2}..., a_{4j}}\) liczby \(\displaystyle{ a^n}\)jest \(\displaystyle{ \neq 0}\) .
Skoro tak, to kazdy ciag:
\(\displaystyle{ a_2, a_3, a_4}\)
\(\displaystyle{ a_5, a_6, ...a_{16}}\)
................
\(\displaystyle{ a_{4^k+1}, a_{4^k+2}, ...,a_{4^{k+1}}}\)
zawiera wyraz niezerowy. \(\displaystyle{ k=[log_4 n] +1}\),
skoro sa to ciagi parami rozłaczne i zawieraja
cyfry liczby \(\displaystyle{ a^n}\).
tj \(\displaystyle{ \{\{a^n\}\} \geq k+1 =log_4 n}\)
finito
Ostatnio zmieniony 4 gru 2007, o 00:04 przez mol_ksiazkowy, łącznie zmieniany 1 raz.
Piotr Rutkowski
Użytkownik
Użytkownik
Posty: 2234
Rejestracja: 26 paź 2006, o 18:08
Płeć: Mężczyzna
Lokalizacja: Warszawa
Podziękował: 22 razy
Pomógł: 390 razy

[Ciągi][Analiza] Niestandardowa granica

Post autor: Piotr Rutkowski »

mol_ksiazkowy pisze::arrow: Jeśli \(\displaystyle{ 1 \leq j \leq \frac{n}{4}}\) , to co najmniej jedna z cyfr \(\displaystyle{ a_{j+1}, a_{j+2}..., a_{4j}}\) liczby \(\displaystyle{ a^n}\) jest \(\displaystyle{ \neq 0}\)
Mógłbyś przytoczyć dowód tego?
Awatar użytkownika
arek1357
Użytkownik
Użytkownik
Posty: 5748
Rejestracja: 6 gru 2006, o 09:18
Płeć: Mężczyzna
Lokalizacja: blisko
Podziękował: 131 razy
Pomógł: 526 razy

[Ciągi][Analiza] Niestandardowa granica

Post autor: arek1357 »

Zaraz zaraz czy może ja źle zrozumiałem w lemacie a jest liczbą parzystą
a trójka jest liczbą nieparzystą więc jak lemat ma się do zadania???
Piotr Rutkowski
Użytkownik
Użytkownik
Posty: 2234
Rejestracja: 26 paź 2006, o 18:08
Płeć: Mężczyzna
Lokalizacja: Warszawa
Podziękował: 22 razy
Pomógł: 390 razy

[Ciągi][Analiza] Niestandardowa granica

Post autor: Piotr Rutkowski »

Znaczy natknąłem się w międzyczasie na podobne rozwiązanie. Rozwiązanie mola_książkowego jest lematem zacytowanym dosłownie z książki Browkina Zadania z olimpiad Tom 5. Podane tam zadanie dotyczyło akurat liczby 1987 (bodajże). Co do liczb nieparzystych niepodzielnych przez 5 został dodany komentarz, że dowód tego prowadzi się analogicznie.
Pozdrawiam
polskimisiek
Awatar użytkownika
mol_ksiazkowy
Użytkownik
Użytkownik
Posty: 11409
Rejestracja: 9 maja 2006, o 12:35
Płeć: Mężczyzna
Lokalizacja: Kraków
Podziękował: 3155 razy
Pomógł: 748 razy

[Ciągi][Analiza] Niestandardowa granica

Post autor: mol_ksiazkowy »

polskimisiek napisa/l;
Znaczy natknąłem się w międzyczasie na podobne rozwiązanie. Rozwiązanie mola_książkowego jest lematem zacytowanym dosłownie z książki Browkina Zadania z olimpiad Tom 5. Podane tam zadanie dotyczyło akurat liczby 1987 (bodajże).
prof A Schinzel jest autorem tego rozw, ja tylko zacytowałem, zadanie tego typu wedrowalo od czasu do czasu po roznych olimpiadach, W przypadku o jaki tu mowa, gdy s(n) jest suma cyfr w zapisie l. n, to skoro co wynika z regul zwyklego dodawania "w slupku" mialem pomysl aby uzyc, szacowania
\(\displaystyle{ s() \leq s(a)+s(b)}\), tj
\(\displaystyle{ s(11*3^n) \leq s(10*3^n)+s(3^n)=2S(3^n)}\),
zas liczba \(\displaystyle{ s(11*3^n)}\), na mocy dodawania w słupku tez powinna rosnac do niskonczonosci wraz z n, ale to taka idea, bez dowodu...Mam tez inna-moze ktos dowoiedzie lub obali, iz w zapisie dziesietnym \(\displaystyle{ s(3^n)}\), nie wystepuja obok siebie dwa zera, tj to by znaczyło iz skoro jest to podzielne przez 9, co ciag ten rosnie bardzo szybko ..., tak wiec same poszlaki, zadnych dowodów
exupery
Użytkownik
Użytkownik
Posty: 518
Rejestracja: 21 lut 2007, o 17:51
Płeć: Mężczyzna
Lokalizacja: Kluczewsko
Podziękował: 20 razy
Pomógł: 67 razy

[Ciągi][Analiza] Niestandardowa granica

Post autor: exupery »

a nie można, by rozwiązać w ten sposób:
Q-liczba cyfr
maksymalnie dla 3 kolejnych n Q jest stałe, dla kolejnego n Q zwiększa się o 1. Czyli z tego faktu wynika, że przy n dążącym do nieskończoności Q dąży do nieskończoności, a skoro Q dąży do nieskończoności to \(\displaystyle{ \lim_{n\to\infty} \{\{\ 3^{n }\}\}\ =+ }\)
Awatar użytkownika
przemk20
Użytkownik
Użytkownik
Posty: 1094
Rejestracja: 6 gru 2006, o 22:47
Płeć: Mężczyzna
Lokalizacja: Olesno
Podziękował: 45 razy
Pomógł: 236 razy

[Ciągi][Analiza] Niestandardowa granica

Post autor: przemk20 »

heh, tylko tu chodzi o sume cyfr
exupery
Użytkownik
Użytkownik
Posty: 518
Rejestracja: 21 lut 2007, o 17:51
Płeć: Mężczyzna
Lokalizacja: Kluczewsko
Podziękował: 20 razy
Pomógł: 67 razy

[Ciągi][Analiza] Niestandardowa granica

Post autor: exupery »

przemk20 pisze:heh, tylko tu chodzi o sume cyfr

Owszem, ale skoro liczba tych cyfr dąży do nieskończoności to ich suma też dąży do nieskończoności, chociaż tutaj wypadałoby jeszcze rozważyć sytuacje z pojawianiem się zer w tej liczbie.
Awatar użytkownika
limes123
Użytkownik
Użytkownik
Posty: 666
Rejestracja: 21 sty 2008, o 22:48
Płeć: Mężczyzna
Lokalizacja: Ustroń
Podziękował: 26 razy
Pomógł: 93 razy

[Ciągi][Analiza] Niestandardowa granica

Post autor: limes123 »

No wlasnie i tu jest problem
ironleaf
Użytkownik
Użytkownik
Posty: 66
Rejestracja: 31 gru 2008, o 19:07
Płeć: Mężczyzna
Lokalizacja: Wojsławice
Podziękował: 4 razy
Pomógł: 8 razy

[Ciągi][Analiza] Niestandardowa granica

Post autor: ironleaf »

...a istnieją potęgi trójki postaci 1000...[jakiś blok cyfr]...0001, przy czym można żądać dowolnie wielu zer. Tego dowodzi się z tw. Eulera i Weyla-Sierpińskiego.
Awatar użytkownika
XMaS11
Użytkownik
Użytkownik
Posty: 382
Rejestracja: 6 mar 2008, o 21:40
Płeć: Mężczyzna
Lokalizacja: Suchedniów/Kielce
Podziękował: 5 razy
Pomógł: 47 razy

[Ciągi][Analiza] Niestandardowa granica

Post autor: XMaS11 »

Ok, może ktoś powiedzieć czy to jest ok?
Niech \(\displaystyle{ S(n)}\) oznacza sumę cyfr liczby \(\displaystyle{ n}\) w zapisie dziesiętnym.
Pokażemy, że dla każdego \(\displaystyle{ k}\) istnieje \(\displaystyle{ l}\) takie, że \(\displaystyle{ S(3^l) \geqslant S(3^k)+1}\).
Weźmy \(\displaystyle{ m}\) takie, że \(\displaystyle{ 10^m >3^k}\) Z tw. Eulera zachodzi:
\(\displaystyle{ 10^m|(3^k)^{\varphi(10^m)} - 1}\) Czyli :
\(\displaystyle{ 10^m|(3^k)^{\varphi(10^m)+1} - 3^k}\)
Ponieważ \(\displaystyle{ 10^m>3^k}\), to \(\displaystyle{ (3^k)^{\varphi(10^m)+1}}\) kończy się cyframi, które tworzą liczbę \(\displaystyle{ 3^k}\), oczywiście więc \(\displaystyle{ S((3^k)^{\varphi(10^m)+1})\geqslant S(3^k)+1}\), co załatwia sprawe?
ironleaf
Użytkownik
Użytkownik
Posty: 66
Rejestracja: 31 gru 2008, o 19:07
Płeć: Mężczyzna
Lokalizacja: Wojsławice
Podziękował: 4 razy
Pomógł: 8 razy

[Ciągi][Analiza] Niestandardowa granica

Post autor: ironleaf »

Nie widzę sposobu dokończenia. Patrz definicja granicy i występujący w niej kwantyfikator 'dla każdego'.
kuba958
Użytkownik
Użytkownik
Posty: 74
Rejestracja: 24 mar 2009, o 17:29
Płeć: Mężczyzna
Lokalizacja: Szczecinek
Podziękował: 1 raz
Pomógł: 13 razy

[Ciągi][Analiza] Niestandardowa granica

Post autor: kuba958 »

Wyjdźmy z oczywistego faktu, iż \(\displaystyle{ \lim_{n \to \infty}3^{n}=+\infty}\). Wynika stąd, że istnieje nieskończenie wiele wyrazów ciągu \(\displaystyle{ 3^{n}}\) większych od np. \(\displaystyle{ 10^{D}}\), gdzie D jest dowolną liczbą rzeczywistą.
Zauważmy, że suma cyfr każdego z tych wyrazów jest większa od D, gdyż każdy z tych wyrazów w zapisie dziesiętnych posiada co najmniej D+1 cyfr. Istnieje zatem nieskończenie wiele wyrazów ciągu {{\(\displaystyle{ 3^{n}}\)}} większych od D.

Stąd \(\displaystyle{ \lim_{n \to \infty}}\){{\(\displaystyle{ 3^{n}}\)}}\(\displaystyle{ =+\infty}\)
Awatar użytkownika
mol_ksiazkowy
Użytkownik
Użytkownik
Posty: 11409
Rejestracja: 9 maja 2006, o 12:35
Płeć: Mężczyzna
Lokalizacja: Kraków
Podziękował: 3155 razy
Pomógł: 748 razy

[Ciągi][Analiza] Niestandardowa granica

Post autor: mol_ksiazkowy »

No ale moge sie zera przeplatac w sposob dosc gęsty- tego nie wiedomo, ...
XMas11 podciag silnie rosnacy, jeszcze malo.
ODPOWIEDZ